Physics Updated JAMB Practice Questions & Answers 2021 Set 2

Physics JAMB Practice Questions & Answers 2021 Set 2

1. The limiting frictional force between two surface depends on

I. the normal reaction between the surfaces

II. the area of surface in contact

III. the relative velocity between the surfaces

IV. the nature of the surface

  • A. I only
  • B. I & IV only
  • C. II only
  • D. III only
Correct Answer: Option B
Explanation

- Friction depends on the nature of the surfaces in contact

- Solid friction is independent of the area of the surfaces in contact and the relative velocity between the surfaces.

    2. If a body moves with a constant speed and at the same time undergoes an acceleration, its motion is said to be

    • A. oscillation
    • B. circular
    • C. rotational
    • D. rectilinear
    Correct Answer: Option B
    Explanation

    Circular motion has uniform

      3. When blue and green colours of light are mixed, the resultant colour is

      • A. cyan
      • B. magenta
      • C. black
      • D. yellow
      Correct Answer: Option A
      Explanation

      Cyan is a bluish-green colour

        4.A metal rod has a length of 100cm at 200oC. At what temperature will its length be 99.4cm. If the linear expansivity of the material of the rod is 2 x 105C1

        • A. 200oC
        • B. 300oC
        • C. 100oC
        • D. -100o

        Correct Answer: Option D
        Explanation

        L1

        = 100cm, T1 = 200oC

        L2

        = 99.4cm,T2 = ?

        α

        = 2 x 105C1 α

        L2L1L1(T2T1) 2 x 105=10099.4100(200T2)=0.6100(200T2)

        2 x 105

        100(200T2)=0.6 200T2=0.60.002=300oC

        T2=200300=100oC

        5. According to kinetic molecular model, in gases

        • A. The molecule are very fast apart & occupy all the space made available
        • A. The particles vibrate about fixed positions and are held together by the strong intermolecular bond between them
        • B. The particles occur in clusters with molecules slightly farther apart
        • D. The particles are closely packed together, they occupy minimum space & are usually arranged in a regular pattern 

        Correct Answer: Option A
        Explanation

        In kinetic molecular model, gases are energised and thus moves freely, fast as they occupy specific space

        6

        The value of T in the figure above is

        • A. 30N
        • B. 10.0N
        • C. 20N
        • D. 40N 

          Correct Answer: Option D
          Explanation

          Tsin30 + Tsin30 =40

          2Tsin30 = 40

          Tsin30 = 40/2 = 20

          T(12) = 20

          T = 20 x 2 = 40N

          7. A train has an initial velocity of 44m/s and an acceleration of -4m/s2

          . Calculate its velocity after 10 seconds

          • A. 10m/s
          • B. 6m/s
          • C. 8m/s
          • D. 4m/s 

            Correct Answer: Option D
            Explanation

            V = u _ at, at u = 44m/s

            44 + [-4]10

            4m/s

            8. Lamps in domestic lightings are usually in

            • A. series
            • B. divergent
            • C. convergent
            • D. parallel 
            Correct Answer: Option D
            Explanation

            Lamps in lightenings are arranged to reduce the heat losses

            - resultant resistance is small for parallel arrangement which indicate low heat losses [Heat α R]

            9. During the transformation of matter from the solid to the liquid state, the heat supplied does not produce temperature increase because

            • A. all the heat is used to break the bonds holding the molecules of the solid together
            • B. the heat capacity has become very large as the substance melts
            • C. the heat energy is quickly conducted away
            • D. the heat gained is equal to the heat lost by the substance 

              Correct Answer: Option A
              Explanation

              During the transformation of the matter from the solid to the liquid state, the heat supplied does not produce a temperature increase because all the heat is used to break the bonds holding the molecules of the solid together

              10. In a slide wire bridge, the balance is obtained at a point 25cm from one end of wire 1m long. The resistance to be tested is connected to that end and a standard resistance of 3.6Ω

              is connected to the other end of the wire. Determine the value of the unknown resistance

                • A. 3.2Ω
              • B. 1.4Ω
              • C. 3.21Ω
              • D. 1.2Ω
                 
                Correct Answer: Option D
                Explanation

                R3.6=7525=13
                3R = 3.6
                R = 1.2Ω

              11. Electrons were discovered by

              • A. Dalton
              • B. James Charwick
              • C. J.J. Thompson
              • D. Niels Bohr 

                Correct Answer: Option C
                Explanation

                Electron was discovered J.J Thompson in 1879

                12. Which of the following is/are the limitations to the Rutherford's atomic models?

                I. It is applicable when energy is radiated as electrons are revolving

                II. It is applicable when energy is radiated in a continuous mode

                III. It is applicable to an atom with only one electron in the other shell

                • A. I only
                • B. II only
                • C. I & II only
                • D. I, II & III only

                Correct Answer: Option C
                Explanation

                Rutherford assumed that (I) energy is radiated when electrons are revolving (II) energy is radiated in a continuous mode. These are limitations of Rutherford's model

                  13. Which of the following equations is the correct definition of the reactance of an indicator L?

                  A. Reactance = (Amplitude of voltage) ÷ (Amplitude of current)

                  • B. Reactance = (Amplitude of voltage) x (Amplitude of current)
                  • C. Reactance = (Amplitude of current)2÷(Amplitude of voltage) 
                  • D. Reactance = (Amplitude of current)2÷ (Amplitude of voltage)12

                  Correct Answer: Option A
                  Explanation

                  Reactance is the value of an indicator or a capacitor in ohms

                  14. Ripple in a power supply unit is caused by

                  • A. using an alternating current source
                  • B. forward voltage drop
                  • C. heavy load
                  • D. using a zener diode
                  Correct Answer: Option D
                  Explanation

                  The correct option is "Using a zener diode" as fluctuation of d.c signal results from the rectification of a.c to d.c.

                    15. When th temperature of a liquid is increased, its surface tension

                    • A. increases
                    • B. decreases
                    • C. remain constant
                    • D. increases then decreases 
                    Correct Answer: Option B
                    Explanation

                    Surface tension or elasticity of a fluid decreases with increased in temperature

                      16. The distance between an object and its real image in a convex lens is 40cm. If the magnification of the image is 3, calculate the focal length of the lens

                      • A. 6.5 cm
                      • B. 7.5 cm
                      • C. 8.5 cm
                      • D. 4.5 cm 

                      Correct Answer: Option B
                      Explanation

                      u + v = 40

                      vu = 3

                      v = 3u

                      u + 3u = 40

                      4u = 40

                      u = 10cm

                      v = 3u = 30cm

                      f = uvu+v=10(30)10+30=30040

                      = 7.5 cm

                        17. A ray of light passes through the centre of curvature of a concave mirror and strikes the mirror. At what angle is the ray reflected?

                        • A. 180o

                        • B. 90o
                        • C. 0o
                        • D. 60o

                        Correct Answer: Option C
                        Explanation

                        - a ray of light passing through the centre of a curvature is returned along the same path

                        - angle difference along the same path is zero

                        18. The pin-hole camera produces a less sharply defined image when the

                        • A. pin-hole is larger
                        • B. illumination is less
                        • C. screen is further from the pin-hole
                        • D. object is further from the pin-hole
                        Correct Answer: Option A
                        Explanation

                        - a less sharp or blurred image is due to increased brightness

                        - increase brightness is also due to large hole or many pin holes

                          19. A straight wire 15cm long, carrying a current of 6.0A is in a uniform field of 0.40T. What is the force on the wire when it is at right angle to the field

                          • A. 0.46N
                          • B. 0.35N
                          • C. 0.36N
                          • D. 0,24N 

                          Correct Answer: Option C
                          Explanation

                          F = BIL = 0.4 x 6 x 15100

                          = 0.36N

                            20. The pitch of a screw jack is 0.45cm and the arm is 60cm long. If the efficiency of the Jack is 75/π %, calculate the mechanical advantage.

                            • A. 400
                            • B. 300
                            • C. 200
                            • D. 150 

                            Correct Answer: Option C
                            Explanation

                            P = 0.45cm, L = 60cm, Eff = 75/π%

                            VR

                            (Screw

                            system)

                            =

                            2πr    


                            P

                            =

                            2πL    


                            P

                            M.A

                            =

                            Eff% × VR    


                            100

                            =

                            75    


                            π

                            ×

                            1


                            100

                            ×

                            2π × 60    


                            0.45

                            =

                            75 × 800    


                            300

                            =

                            200

                              21. When the downward current flows in a straight vertical conductor, the direction of its magnetic field at a point due north of the wire is


                              • A. Upward
                              • B. North
                              • C. South
                              • D. West 

                              Correct Answer: Option D
                              Explanation

                              At a point due N of the wire, the field is due east, at a point due S of the wire, the field is due west.

                                22. Aluminium is sometimes used as the leaf of an electroscope because it

                                • A. is a light material
                                • B. is a good conductor
                                • C. is a good insulator
                                • D. can be converted into thin sheets
                                Correct Answer: Option D
                                Explanation

                                - Aluminium can be made in thin sheet like Gold.

                                - the leaf is a thin material that can be diverged easily.

                                  23

                                  The diagram above represents the stress-strain graph of a loaded wire. Which of these statements is correct?

                                  • A. At J, the wire becomes plastic
                                  • B. J is the yield point
                                  • C. L is the elastic limit
                                  • D. At K, the wire breaks 

                                  Correct Answer: Option B
                                  Explanation

                                  - I is the elastic limit

                                  - the end of the constant part J is the yield point

                                  - L is the break point.

                                    24. A supply of 400V is connected across capacitors of 3μf and 6μf in series. Calculate the charge

                                    • A. 8 x 104C
                                    • B. 4 x 102C
                                    • C. 8 x 103C
                                    • D. 4 x 108

                                    Correct Answer: Option A
                                    Explanation

                                    CT

                                    =

                                    C1 × C2    


                                    C1 + C2

                                    =

                                    3 × 6    


                                    3 + 6

                                    189 = 2μf

                                    Q = CV

                                    ⇒ 2 × 106 × 400

                                    ⇒ 800 × 106C = 8 × 104C

                                    25. A vibrator causes water ripples to travel across the surface of a tank. The wave travels 50cm in 20s and the distance between successive crests is 5cm. Calculate the frequency of the vibrator

                                    • A. 5Hz
                                    • B. 6Hz
                                    • C. 5.2Hz
                                    • D. 25Hz 

                                     

                                    Correct Answer: Option A
                                    Explanation

                                    V

                                    =

                                    distance    


                                    time

                                    =

                                    50 cm    


                                    25

                                    =

                                    25 cm/s

                                    V = fλ

                                    f

                                    =

                                    V.λ

                                    =

                                    25    


                                    5

                                    =

                                    5Hz

                                    26. A thermocouple thermometer is connected to a millivoltmeter which can read up to 10mV. When one junction is in ice at 0°C and the other is steam at 100°C, the millivoltmeter reads 4mV. What is the maximum temperature which this arrangement can measure

                                    A. 100oC

                                    • B. 248oC
                                    • C. 250oC
                                    • D. 350o
                                      Correct Answer: Option C
                                      Explanation
                                      tθ
                                      - to = 10,
                                      t100 - to = 4

                                    - to


                                    t100 - to

                                    θ

                                    =

                                    tθ

                                    ×

                                    100

                                    10    


                                    4

                                    ×

                                    100

                                    =

                                    250°C

                                     

                                    27. An alternating current can induce voltage because it has

                                    • A. ripple value
                                    • B. varying magnetic field
                                    • C. weaker magnetic field than direct current
                                    • D. high peak value 

                                    Correct Answer: Option B
                                    Explanation

                                    As induced voltage is produced by varying magnetic field. The induced voltage is proportional to the rate of change of magnetic flux linking the coil.

                                      28. The limiting frictional force between two surfaces depends on

                                      I. the normal reaction between the surfaces

                                      II. the area of surface in contact

                                      III. the relative velocity between the surfaces

                                      IV. the nature of the surfaces

                                      • A. I only
                                      • B. II only
                                      • C. III only
                                      • D. I & IV only 

                                      Correct Answer: Option D
                                      Explanation

                                      - Friction depends on the nature of the surfaces in contact

                                      - Solid friction is independent of the area of the surfaces in contact and the relative velocity between the surfaces.

                                        29. During the transformation of matter from the solid to the liquid state, the heat supplied does not produce a temperature increase because

                                        • A. the heat energy is quickly conducted away
                                        • B. the heat capacity has become very large as the substance melts
                                        • C. the heat gained is equal to the heat lost by the substance
                                        • D. all the heat is used to break the bonds holding the molecules of the solid together 

                                        Correct Answer: Option D
                                        Explanation

                                        During the transformation of matter from the solid to the liquid state, the heat supplied does not produce a temperature increase because all the heat is used to break the bonds holding the molecules of the solid together

                                          30. The statement 'Heat lost by the hot body equals that gained by the cold one' is assumed when determining specific that heat capacity by the method of mixtures. Which of the following validates the assumption?

                                          I. Lagging the Calorimeter

                                          II. Ensuring that only S.I units are used

                                          III. Weighing the calorimeter, the lid and the stirrer.

                                          • A. I only
                                          • B. I & II only
                                          • C. I & III only
                                          • D. II & III only 

                                          Correct Answer: Option C
                                          Explanation

                                          I & III only as units are independent of energy equation

                                          31. Which of the following graphs is consistent with charle's law?

                                          I.

                                          II.

                                          III.

                                          IV.

                                          • A. I
                                          • B. II
                                          • C. III
                                          • D. IV 

                                          Correct Answer: Option D
                                          Explanation

                                          as V α T and temperature is in °C rather than kelvin

                                            32. In the molecular explanation of conduction, heat is transferred by the

                                            • A. Free electrons
                                            • B. Free atoms
                                            • C. Free molecules
                                            • D. Free solids 

                                            Correct Answer: Option A
                                            Explanation

                                            - Conduction is explained in terms of the free electrons.

                                            - Convention is explained in terms of the movement of the fluid involved.

                                            - Radiation is explained in terms of invisible electromagnetic wave

                                              33. The conductivity of gases at low pressure can be termed as

                                              I. hot cathode emission

                                              II. thermo ionic emission

                                              III. cold cathode emission

                                              IV. Field emission

                                              • A. I & II
                                              • B. II & III
                                              • C. III & IV
                                              • D. IV & V
                                              Correct Answer: Option D
                                              Explanation

                                              - sound travels through solids, liquids & gases

                                              - sound travels faster in solid than in liquid & faster in liquid than in gases.

                                                34. Which of the following media allow the transmission of sound waves through them?

                                                I. air

                                                II. liquid

                                                III. solids

                                                • A. I & II only
                                                • B. I & III only
                                                • C. II & III only
                                                • D. I, II & III
                                                Correct Answer: Option D
                                                Explanation

                                                - sound travels through solids, liquids & gases

                                                - sound travels faster in solid than in liquid & faster in liquid than in gases.

                                                  35. The earth's gravitational field intensity at its surface is about

                                                  (G = 6.7 × 1011

                                                  Nm2/kg2, mass of the earth is 6 × 1024kg, radius of the earth is 6.4 × 106m, g on the earth = 9.8m/s2

                                                  )

                                                  • A. 6.7N/kg
                                                  • B. 7.9N/kg
                                                  • C. 8.0N/kg
                                                  • D. 9.8N/kg 

                                                  Correct Answer: Option D
                                                  Explanation

                                                  E

                                                  =

                                                  F


                                                  m

                                                  =

                                                  GMm/d2


                                                  m

                                                  =

                                                  GM    


                                                  d2

                                                  E

                                                  =

                                                  6.7 × 1011

                                                  × 6 × 1024


                                                  (6.4 × 106)2

                                                  =

                                                  9.8N    


                                                  kg

                                                  E = g = 9.8N/kg

                                                    36. The following are parts of the eye

                                                    I. Retina

                                                    II. Pupil

                                                    III. Iris

                                                    The correct equivalent in the camera in the same order are

                                                    • A. Diaphragm, Aperture, film
                                                    • B. Aperture, Diaphragm, Film
                                                    • C. Film, Diaphragm, Aperture
                                                    • D. Film, Aperture, Diaphragm
                                                    Correct Answer: Option D
                                                    Explanation

                                                    - retina is similar to film

                                                    - pupil is similar to aperture

                                                    - iris is similar to diaphragm

                                                      37. According to kinetic molecular model, in gases

                                                      • A. The particles are closely packed together, they occupy minimum space & are usually arranged in a regular pattern
                                                      • B. The particles occur in clusters with molecules slightly farther apart
                                                      • C. The molecule are very fast apart & occupy all the space made available
                                                      • D. The particle vibrate about fixed positions and are held together by the strong intermolecular bonds between them 

                                                      Correct Answer: Option C
                                                      Explanation

                                                      In kinetic molecular model, gases are energized and thus moves freely, fast as they occupy specific space

                                                        38. A force of 6N acts horizontally on a stationary mass of 2kg for 4s. The kinetic energy gained by the mass is _________

                                                        Explanation

                                                        F = 6N, m = 2kg, t = 4s, u = 0 (stationary or at rest)

                                                        from rest

                                                        K.E

                                                        =

                                                        1    


                                                        2

                                                        mv2

                                                        F

                                                        =

                                                        mv    


                                                        t

                                                        K.E

                                                        =

                                                        1    


                                                        2

                                                        m(

                                                        Ft    


                                                        m

                                                        )2

                                                        =

                                                        1    


                                                        2

                                                        ×

                                                        2 x (6×42)2=122=144J

                                                        39. The momentum of a car moving at a constant speed in a circular track

                                                        • A. can be both positive and negative
                                                        • B. can be zero depending on its position in the track
                                                        • C. can be zero depending on its position in the track
                                                        • D. a centrifugal
                                                        Correct Answer: Option C
                                                        Explanation

                                                        Movement of an object in a circle with an acceleration towards its center is provided by change in velocity & centripetal force a α V α Fc

                                                          40. A metal rod has a length of 100cm at 200°C. At what temperature will its length be 99.4cm. If the linear expansivity of the material of the nod is 2 × 105C1

                                                          Explanation

                                                          L1

                                                          = 100cm, T1 = 200°C

                                                          L2 = 99.4cm, T2 = ?

                                                          α 2 × 105 C1

                                                          α

                                                          =

                                                          L2

                                                          - L1


                                                          L1(T2 - T1

                                                          )

                                                          2

                                                          ×

                                                          105

                                                          =

                                                          110 - 99.4


                                                          100(200 - T2

                                                          )

                                                          =

                                                          0.6


                                                          100(200 - T2

                                                          )

                                                          2 × 105×100(200T2) = 0.6

                                                          200

                                                          -

                                                          T2

                                                          =

                                                          0.6


                                                          0.002

                                                          =

                                                          300°C

                                                          T2

                                                          = 200 - 300 = -100°C

                                                          41. The lead-acid accumulator consists of

                                                          • A. lead as the positive electrode
                                                          • B. lead acid as the negative electrode
                                                          • C. hydrochloric acid as the electrolyte
                                                          • D. tetraoxosulphate (vi) acid as the electrolyte
                                                          Correct Answer: Option D
                                                          Explanation

                                                          - the positive pole is lead peroxide (PbO2)

                                                          - the negative pole is head

                                                          - the electrolyte is H2SO4

                                                            42. When water is boiling, it

                                                            • A. gets hotter
                                                            • B. increase is mass
                                                            • C. decreases in mass
                                                            • D. changes to steam
                                                            Correct Answer: Option D
                                                            Explanation

                                                            Boiling indicates changes from Liquid to gas.

                                                              43. A mixture of blue and red pigment when illuminated by white light will appear

                                                              • A. green
                                                              • B. orange
                                                              • C. purple
                                                              • D. magenta
                                                              Correct Answer: Option C
                                                              Explanation

                                                              A mixture of blue & red pigments i.e parts is purple or violet

                                                              When illuminated by white light,

                                                              V + ROYGBIV = V = Violet or purple.

                                                                44. A siren having a ring of 200 hole makes 132 rev/min. A jet of air is directed on the set of holes. Calculate the frequency and wavelength in air of the note produced (take v = 350m/s)

                                                                • A. 0.875m
                                                                • B. 0.625m
                                                                • C. 0.335m
                                                                • D. 0.125m 

                                                                Correct Answer: Option A
                                                                Explanation

                                                                n = 200, S = 132 rev/min, v = 350m/s2

                                                                f

                                                                =

                                                                ns

                                                                =

                                                                200

                                                                ×

                                                                132

                                                                rev


                                                                min

                                                                ×

                                                                1min    


                                                                60s

                                                                =

                                                                440Hz

                                                                λ

                                                                =

                                                                v    


                                                                f

                                                                =

                                                                350    


                                                                440

                                                                =

                                                                0.875m

                                                                   

                                                                  45 A man on a bench will exert the greatest pressure on the bench when he

                                                                  • A. lies flat on his back
                                                                  • B. lies flat on his belly
                                                                  • C. stands on both feet
                                                                  • D. stands on the toes of one foot 6 
                                                                    Correct Answer: Option D
                                                                    Explanation

                                                                    P

                                                                    =

                                                                    F    


                                                                    A

                                                                    hence, pressure increases when the areas decreases.

                                                                    46. Gases conduct electricity under

                                                                    • A. high pressure & high voltage
                                                                    • B. low pressure & low voltage
                                                                    • C. normal pressure & low voltage
                                                                    • D. low pressure & high voltage

                                                                    Correct Answer: Option D
                                                                    Explanation

                                                                    Gases conduct electricity under low pressure & high voltage

                                                                    47. Neutrons were discovered by

                                                                    • A. Faraday
                                                                    • B. Dalton
                                                                    • C. James Chadwick
                                                                    • D. Rutherford 

                                                                    Correct Answer: Option C
                                                                    Explanation

                                                                    It was discovered by James Chadwick in 1932.

                                                                    48. The mass of a nucleus is the

                                                                    • A. total number of its proton & neutrons
                                                                    • B. number of its protons
                                                                    • C. mass of its electrons
                                                                    • D. total number of its protons & electrons 

                                                                    Correct Answer: Option A
                                                                    Explanation

                                                                    Nucleus = mass of proton + mass of neutron

                                                                    49. Three resistors with resistance 200Ω, 500Ω and 1kΩ are connected in series. A 6v battery is connected to either end of the combination. Calculate the potential difference between the ends of 200Ω resistance.

                                                                    • A. 0.71V
                                                                    • B. 0.7V
                                                                    • C. 0.82V
                                                                    • D. 0.8V 

                                                                    Correct Answer: Option A
                                                                    Explanation

                                                                    Req = 200 + 500 + 1000 = 1700Ω

                                                                    P.d

                                                                    (across

                                                                    200Ω)

                                                                    =

                                                                    200


                                                                    1700

                                                                    ×

                                                                    6

                                                                    =

                                                                    0.71V

                                                                    50. An a.c of 1A at a frequency of 800 cycles per second flows through a coil, the inductance of which is 2.5mH and the resistance of which is 5Ω. What is the power absorbed in the Coil?

                                                                  • A. 5W
                                                                  • B. 6W
                                                                  • C. 7W
                                                                  • D. 8W 

                                                                  Correct Answer: Option A
                                                                  Explanation

                                                                  I = 1A, F = 800 cycles/s = 800Hz

                                                                  R = 5Ω, L = 2.5mH

                                                                  P = I2

                                                                  R = I2 × 5 = 5W

                                                                  51

                                                                  In semi-conductor, the carriers of current at room temperature are

                                                                  • A. electrons only
                                                                  • B. electrons and holes
                                                                  • C. holes only
                                                                  • D. electron and ions 

                                                                  Correct Answer: Option B
                                                                  Explanation

                                                                  Electron and holes represents the negative and positive charge respectively.

                                                                  52. Water and Kerosine are drawn respectively into the two limbs of a Hare's apparatus. The destiny of water is 1.0gcm3 and the density of kerosine is 0.80gcm3

                                                                  . If the height of the water column is 20.0cm, calculate the height of the kerosine column.

                                                                  • A. 1.6cm
                                                                  • B. 16.0cm
                                                                  • C. 20.0cm
                                                                  • D. 25.0cm 

                                                                  Correct Answer: Option D
                                                                  Explanation

                                                                  Devices with different liquids

                                                                  d1h1 = d2h2

                                                                  1 × 20 = 0.8 × h

                                                                  h

                                                                  =

                                                                  20


                                                                  0.8

                                                                  =

                                                                  25cm

                                                                  53. When a girl moves towards a plane mirror at a speed of 4.0m/s, the distance between the girl & her image reduces a speed of

                                                                  • A. 2 m/s
                                                                  • B. 4 m/s
                                                                  • C. 8 m/s
                                                                  • D. 12 m/s 

                                                                  Correct Answer: Option C
                                                                  Explanation

                                                                  v

                                                                  =

                                                                  d    


                                                                  t

                                                                  or

                                                                  v

                                                                  α

                                                                  d

                                                                  d = x, v = 4m/s

                                                                  d = 2x, v = ? (girl & image)

                                                                  v

                                                                  =

                                                                  2 × 4    


                                                                  x

                                                                  =

                                                                  8

                                                                  m    


                                                                  s

                                                                  54. One newton × One meter equals?

                                                                  • A. One water
                                                                  • B. One Joule
                                                                  • C. One ampere
                                                                  • D. One kilogram 

                                                                  Correct Answer: Option B
                                                                  Explanation

                                                                  1N × 1m = 1Nm = 1 Joul (F × d = Work/Energy)

                                                                  55. A microscope is focused on a mark on a table, when the mark is covered by a plate of glass 2m thick, the microscope has to be raised 0.67cm for the mark to be once more in focus. Calculate the refractive index.

                                                                  • A. 1.52
                                                                  • B. 1.68
                                                                  • C. 2.58
                                                                  • D. 1.82

                                                                    Correct Answer: Option A
                                                                    Explanation

                                                                    R = th = 2cm, d = 0.67cm

                                                                    n

                                                                    =

                                                                    R    


                                                                    A

                                                                    =

                                                                    R


                                                                    R.d

                                                                    =

                                                                    2


                                                                    2-0.67

                                                                    =

                                                                    1.52

                                                                    56. Radio waves belongs to the class of ware whose velocity is about

                                                                    • A. 340m/s 
                                                                    •  B. 3 × 106 m/s

                                                                  • C. 3 × 108m/s
                                                                  • D. 3 × 108m/s

                                                                  Correct Answer: Option D
                                                                  Explanation

                                                                  3 × 108 m/s

                                                                  57. A single force which produces the same effect as a set of forces acting together at a point is known as the

                                                                  • A. component
                                                                  • B. equilibrant
                                                                  • C. resultant
                                                                  • D. resistant 

                                                                  Correct Answer: Option C
                                                                  Explanation

                                                                  Resultant is the representing vector for multiple forces.

                                                                  58

                                                                  Which of the following statement about the electromagnet shown above is correct?

                                                                  • A. when the current flows from A to B, the end A will be in north pole
                                                                  • B. when the current flows from A to B, the end A will be south pole
                                                                  • C. when the current flows from A to B, the end B will be north pole
                                                                  • D. none of the above 

                                                                  Correct Answer: Option B
                                                                  Explanation

                                                                  A - B = S - N.

                                                                  Also, starting end of the current is south while terminating end is North.

                                                                  59. Calculate the velocity ratio of a screw jack of pitch 0.2cm if the length of the tommy bar is 23cm

                                                                  • A. 230π

                                                                  • B. 23π
                                                                  • C. 70π
                                                                  • D. 140π

                                                                  Correct Answer: Option A
                                                                  Explanation

                                                                  P = 0.2cm, L = r = 23cm

                                                                  VR

                                                                  =

                                                                  2πr    


                                                                  P

                                                                  =

                                                                  2πL    


                                                                  P

                                                                  =

                                                                  2π×23    


                                                                  0.2

                                                                  =

                                                                  230π

                                                                  60.The Earth's magnetic equator passes through Jos in Nigeria. At Jos, the

                                                                  • A. angle of variation is zero
                                                                  • B. magnetic declination is 90
                                                                  • C. angle of dip is zero
                                                                  • D. horizontal component at the Earth's field is zero 

                                                                  Correct Answer: Option C
                                                                  Explanation

                                                                  - angle of dip is zero at the magnetic equator

                                                                  - angle of variation is the same as angle of declination.



                                                                  Explanation

                                                                  N = 1sinθ

                                                                    = 1sin34

                                                                    = 1.79.

                                                                  Related Posts

                                                                  Subscribe Our Newsletter

                                                                  0 Comments to "Physics Updated JAMB Practice Questions & Answers 2021 Set 2"

                                                                  Post a Comment